Which recrusive formula can be used to generate the sequence shown, where f(1)=5 and n>=1 5,-1,-7,-13,-19

Answers

Answer 1

Answer:

a_n = 11 - 6n

Step-by-step explanation:

you can observe every next element is smaller then the previous one by 6  

a_n = 5 - 6*(n-1)

a_n = 5 - 6n + 6

a_n = 11 - 6n


Related Questions

what is this expression in simplest form.(-11/2x+3)-2(-11/4x-5/2)​

Answers

Answer:

The simplest form of the given expression is 8.

Step-by-step explanation:

(-11/2x + 3) - 2(-11/4x - 5/2)

Distribute 2 to (-11/4x - 5/2)

(-11/2x + 3) - (-11/2x - 5)

Now, combine like terms. The terms with the x value will cancel each other out because a negative plus a positive of the same number will equal zero. For example, -2 + 2 = 0.

So, the expression in the simplest form is going to be 8. The x values have cancelled each other out so all there is left is the constant number which is 8.

Victor owes a total of $44.00 to 4 different friends. If Victor owes each friend an equal amount, which
answer choice represents the change in Victor's money when he pays one friend?

Answers

Answer:

11

Step-by-step explanation:

Let x be the amount of money he owes his friends each.

4x = 44

Divide by 4 on each side

x = 11

Two trees are growing in a clearing. The first tree is 17 feet tall and casts a 10 foot shadow. The second tree casts a 35 foot shadow. How tall is the
second tree to the nearest tenth of a foot?

Answers

Answer:

59.5 feet

Step-by-step explanation:

The second tree is 59.5 feet tall.

Given

Two trees are growing in a clearing.

The first tree is 17 feet tall and casts a 10-foot shadow.

The second tree casts a 35-foot shadow.

Let x be the tall is the second tree.

Then,

The ratio of the height of the tree is;

[tex]\rm \dfrac{17}{10} = \dfrac{x}{35}\\\\17 \times 35 = x \times 10\\\\595 = 10x\\\\x = \dfrac{595}{10}\\\\x = 59.5 \ feet[/tex]

Hence, the second tree is 59.5 feet tall.

To know more about Ratio click the link given below.

https://brainly.com/question/8677748

Which of the following steps can be performed so that the square root property may easily be applied to 2x2 = 16? (1 point)
1. The square root property requires a quantity squared by itself on one side of the equation. The only quantity squared is 16, so divide both sides by 2 before applying the square root property.
2. The square root property requires a quantity squared by itself on one side of the equation. The only quantity squared is x, so divide both sides by 2 before applying the square root property.
3. The square root property requires a quantity squared by itself on one side of the equation. The only quantity squared is x, so divide both sides by 16 before applying the square root property.
4. The square root property requires a quantity squared by itself on one side of the equation. The only quantity squared is 16, so divide both sides by 16 before applying the square root property.​

Answers

Answer:

[tex]\Large \boxed{\mathrm{Option \ 2}}[/tex]

Step-by-step explanation:

[tex]2x^2 =16[/tex]

The square root property requires a quantity squared by itself on one side of the equation. The only quantity squared is x, so divide both sides by 2 before applying the square root property.

The x variable should be isolated on one side of the equation. The x variable is squared so before performing the square root property where we take the square root of both sides, we divide both sides by 2, then take the square root of both sides.

Dividing both sides by 2.

[tex]\displaystyle \frac{2x^2 }{2} =\frac{16}{2}[/tex]

[tex]x^2 =8[/tex]

Taking the square root of both sides.

[tex]\sqrt{x^2 } =\pm \sqrt{8}[/tex]

[tex]x=\pm 2\sqrt{2}[/tex]

Answer:

Option 2

Step-by-step explanation:

2x² = 16

Since x is completely squared and and 2 isn't , so we need to divide both sides by 2. As to find x , we need to "isolate" x first so that is why we need to get rid of 2 with the x². We'll divide both sides by 2 in order to get rid of 2.

Dividing both sides by 2

=> x² = 8

Now , that is the x is isolated so we'll take square root on both sides

=> x = ±2√2

Consider the expression
-4b + 8c + 12 - 8b - 2c + 6.
Part B
What is the value of the expression when b = 2 and c = -3?

Answers

Answer:

8

Step-by-step explanation:

-4(2)+8(-3)+12-8(2)-2(-3)+6=-8-24+12-16+6+6=8

Answer:

-24

Step-by-step explanation:

-4b + 8c + 12 - 8b - 2c + 6

Combine like terms

6c+18-12b

Let b=2  c=-3

6(-3) +18-12(2)

-18 +18 -24

-24

What is the coefficient of the variable in the expression 2 - 5x – 4 + 8?
a
-5
b
-4
с
2
d
8

Answers

Answer:

-5

Step-by-step explanation:

The coefficient is the number attached to the variable.

9.2 x 6.07 please answer with the way how you are doing

Answers

Answer:

answer of this question is 55.844

Step-by-step explanation:

may this answer is helpful for you

The sum of 3 consecutive even numbers is 132

pleaase help!!!

Answers

Answer:

42,44,46

Step-by-step explanation:

we first divide the 132 by the 3 which gives 44

since the numbers involved are even numbers and we know even numbers are numbers divisible by 2 we subtract 2 from the 44 and also add 2 to the 44 the get the rest of the two numbers which is from the above explanation

(44-2),44,(44+2)

42,44,46

we can check whether the above numbers are correct by adding to see whether we get 132

42+44+46=132 which are consecutive even numbers

Answer: 42, 44, and 46

Explanation: Make sure you read through the problem carefully so that you recognize that we are dealing with consecutive even numbers here.

So we can represent our numbers as follows.

X ⇒ first even integer

X + 2 ⇒ second even integer

X + 4 ⇒ third even integer

Since their sum is 132, our equation reads x + (x + 2) + (x + 4) = 132.

Solving from here, we find that x = 42.

So x + 2 is 44 and x + 4 is 46.

Does anyone know the answer plz ?

Answers

Answer:

false it would be something like x root 2

Step-by-step explanation:

Please help! Change 3/8 to a decimal fraction.

Answers

Answer:

0.375

Step-by-step explanation:

0.125 x 3 = 0.375

Answer:

0.375

Step-by-step explanation:

3 x 125

8 x 125

=

375

1000

=

0.375

Other sample problem:

5 = 5×125 = 625 = 0.625

8         8×125         1000

Hope this helps, have a good day :)

Solve the system by substitution.
y = -3x + 17
2x + 3y = -5

Answers

Answer:

(8, -7)

Step-by-step explanation:

Hi there!

We are given this system:

y=-3x+17

2x+3y=5

And we want to solve the system

Let's solve it by substitution.

In the first equation, we are given y=-3x+17, which we can substitute -3x+17 as y into the second equation

2x+3(-3x+17)=5

Now do the distributive property

2x+-9x+51=-5

Combine like terms

-7x+51=-5

Subtract 51 from both sides

-7x=-56

Divide both sides by -7

x=8

Now substitute 8 as the value of x in y=-3x+17 to find the value of y

y=-3(8)+17

y=-24+17

y=-7

The answer is x=8, y=-7. If you need it as a point, it's (8, -7)

Hope this helps!

Figure A is a scale image of Figure B. What is the value of x?

Answers

Answer:

x=30

Step-by-step explanation:

to find the value of x you have to find K(constant number).

[tex]\frac{x}{45}=\frac{18}{27}\\\frac{x}{45}=\frac{2}{3}\\ 3(x)=45*2\\3x=90\\x=\frac{90}{3}\\ x=30[/tex]

Answer:

The value of x is 30

Step-by-step explanation:

Step 1: Find the scale factor

[tex]\frac{FA}{F B} =\frac{45}{27}[/tex]

Step 2: Use the scale factor to solve for x

[tex]\frac{45}{27} =\frac{x}{18}[/tex]

[tex](45)(18) = 27x[/tex]

[tex]810 = 27x[/tex]

[tex]x = 30[/tex]

Therefore x = 30

2 pts Question 1 Write an expression to model the phrase: Myles has $635 and is earning $120 each week as a lifeguard. (Use x as your variable) 2 pts Wuestion 2​

Answers

Answer:

The equation is y= 120x+ 635

Step-by-step explanation:

Hey there!!!,

in this problem we are expected to present an equation for the given scenario, and a way out is to model it after the equation of line

i.e y= mx+c

From the problem statement we can see that the constant amount Myles have is $635, and also earnings of $120 weekly.

Comparing the statement withe equation of line we have

  y= 120x+ 635

as "x" is the number of weeks worked and $635 is the constant amount at hand.

Should you need further clarification on this, let me know

A bikes shop has 11 red bikes, 3 blue bikes, 4 orange bikes, and 12 silver bikes. For every 1 orange bike, there are 3_______

Answers

Answer:

[tex]\huge\boxed{Answer=>silver}[/tex]

Step-by-step explanation:

Information given:

1. 11 red bikes

2. 3 blue bikes

3. 4 orange bikes

4. 12 silver bikes

...........................................................

For every 1 orange bike, there are 3 ______

Note: there is 4 orange bikes.

(Denominator will be 4 since 4 orange bikes)

Red: [tex]11/4[/tex]

Blue: [tex]3/4[/tex]

(Don’t include orange)

Silver: [tex]12/4[/tex] (which equals 3)

We can’t divide any of the fractions accept for silver.

So the answer is silver. Hope this helps! ✨by: RobloxYt ✨

6.8 - 4.2b = 5.6b - 3

Answers

Answer: b = 1

Step-by-step explanation:

Given

6.8 - 4.2b = 5.6b - 3

Add 4.2b on both sides

6.8 - 4.2b + 4.2b = 5.6b - 3 + 4.2b

6.8 = 9.8b - 3

Add 3 on both sides

6.8 + 3 = 9.8b - 3 + 3

9.8 = 9.8b

Divide 9.8 on both sides

9.8 / 9.8 = 9.8b / 9.8

[tex]\boxed{b=1}[/tex]

Hope this helps!! :)

Please let me know if you have any questions

B=1 !!!! Hope you got that right

Rona mixes 2 pounds of meat with some chopped vegetables to make a mixture. She divides the mixture into 4 equal portions. Each portion weighs 3 pounds. Which equation and solution shows the total amount of chopped vegetables she used in the mixture?
One-fourth (2 + v) = 3; v = 10 pounds of chopped vegetables
One-fourth (2 + v) = 3; v = 4 pounds of chopped vegetables
4 (2 + v) = 3; v = 1 pound of chopped vegetables
4 (2 + v) = 3; v = 5 pounds of chopped vegetables

Answers

Answer:

First choice: (1/4)(2 + v) = 3; v = 10 pounds of chopped vegetables

Step-by-step explanation:

"2 pounds of meat with some chopped vegetables"

2 + v

"She divides the mixture into 4 equal portions."

(1/4)(2 + v)

"Each portion weighs 3 pounds."

(1/4)(2 + v) = 3

2 + v = 12

v = 10

Answer: (1/4)(2 + v) = 3; v = 10 pounds of chopped vegetables

Answer:

(2+v)=3   v=10

Step-by-step explanation:

Will Mark Brainlest !!!=​

Answers

Answer:

Hello,

Answer m=-1, n=-6,l=2

Step-by-step explanation:

[tex]A*U=B\\\begin{bmatrix}3&-1&0\\0&1&3\\0&0&2\end{bmatrix}*\begin{bmatrix}m\\n\\l\end{bmatrix}=\begin{bmatrix}3\\0\\4 \end{bmatrix}\\\\\\A^{-1}=\begin{bmatrix}\dfrac{1}{3}&\dfrac{1}{3}&\dfrac{-1}{2}\\\\0&1&\dfrac{-3}{2} \\\\0&0&\frac{1}{2}\end{bmatrix}\\\\\\A^{-1}*A*U=A^{-1}*B\\[/tex]

[tex]U=\begin{bmatrix}\dfrac{1}{3}&\dfrac{1}{3}&\dfrac{-1}{2}\\\\0&1&\dfrac{-3}{2}\\\\0&0&\dfrac{1}{2}\end{bmatrix}*\begin{bmatrix}3\\0\\4\\\end{bmatrix}=\begin{bmatrix}-1\\-6\\2\end{bmatrix}\\\\[/tex]

A new site offers a subscription that costs 28.50 for 6 months.what is unit rate price per month? show ur work

Answers

Answer:

The answer is 4.75

Step-by-step explanation:

Since six months is 28.50 then 1 month is equal to x

       28.50: 6 months

           x     : 1 month

 After this  you cross multiple so u divide by 6 both side to get 4.75

   6x/6: 28.50/6

   x=4.75

A ball is dropped from a height of 8 feet .The ball bounce to 80% of its previous height with each bounce how high does the ball bounce on the 5th bounce.

Answers

Answer:

3.2768 feet

Step-by-step explanation:

The situation can be model by h(x)=8*(4/5)^(n-1). For the 5th jump, n=5. The height will be 8*(4/5)^4=3.2768

What is the answer!!

Answers

Answer:

∠ G = 121°

Step-by-step explanation:

The sum of the 3 angles in a triangle = 180°

Sum the 3 given angles and equate to 180

x + 3x + 25 + x - 5 = 180 , that is

5x + 20 = 180 ( subtract 20 from both sides )

5x = 160 ( divide both sides by 5 )

x = 32

Thus

∠ G = 3x + 25 = 3(32) + 25 = 96 + 25 = 121°

Answer:

121

Step-by-step explanation:

if (a+2,b)=(4,5),what is the value of a ?

Answers

Answer:

a = 2

Step-by-step explanation:

(a+2,b)=(4,5)

Since the points are equal we can equate them to find a

Comparing a + 2 to 4

We have

a + 2 = 4

Send 2 to the right side of the equation

a = 4 - 2

We have the final answer as

a = 2

Hope this helps you

Step-by-step explanation:

Here,

according to the question,

(a+2, b)=(4,5)

since, they are equal, equating with their corresponding elements we get,

(a+2)=4

or, a= 4-2

Therefore, the value of a is 2.

Also you can find value of b in same way,

b=5.

Therefore, the value of a abd b are 2 and 5 respectively.

Hope it helps...

Use the slope formula
(19,-16) (-7,15)

Answers

Answer:

-31/26

Step-by-step explanation:

To find the slope

m= ( y2-y1)/(x2-x1)

   = ( 15 - -16)/(-7 - 19)

   = ( 15+16)/(-7-19)

    = 31/ -26

-31/26

A ship travels a distance of 700 km. On the return trip it averages 10km/hr faster and 8 hours less, tp travel the 700km back. Determine how long the original part of the trip took in hours

Answers

Answer:

The total duration of the trip is 48 hours.

Step-by-step explanation:

Let suppose that ship travels at constant speed during its travel. Each stage is represented by the following kinematic equation:

[tex]v =\frac{\Delta s}{\Delta t}[/tex]

Where:

[tex]\Delta s[/tex] - Travelled distance, measured in kilometers.

[tex]\Delta t[/tex] - Time, measured in hours.

[tex]v[/tex] - Speed, measured in kilometers per hour.

Now, each stage is represented by the following expressions:

Outbound trip

[tex]v = \frac{700\,km}{\Delta t}[/tex]

Return trip

[tex]v + 10\,\frac{km}{h} = \frac{700\,kh}{\Delta t - 8\,h}[/tex]

By eliminating [tex]v[/tex] and simplifying the resulting expression algebraically:

[tex]\frac{700\,km}{\Delta t} + 10\,\frac{km}{h} = \frac{700\,km}{\Delta t -8\,h}[/tex]

[tex](700\,km)\cdot \left(\frac{1}{\Delta t - 8\,h}-\frac{1}{\Delta t} \right) = 10\,\frac{km}{h}[/tex]

[tex]\frac{1}{\Delta t - 8\,h}-\frac{1}{\Delta t} = \frac{1}{70}\,\frac{1}{h}[/tex]

[tex]\frac{8\,h}{\Delta t \cdot (\Delta t-8\,h)} = \frac{1}{70}\,\frac{1}{h}[/tex]

[tex]560\,h^{2} = \Delta t\cdot (\Delta t - 8\,h)[/tex]

[tex](\Delta t )^{2}-8\cdot \Delta t - 560 = 0[/tex]

This equation can be solved by means of the Quadratic Formula, whose roots are presented below:

[tex]\Delta t_{1} = 28\,h[/tex] and [tex]\Delta t_{2} = -20\,h[/tex]

Only the first roots offers a physically resonable solution. Then, total duration of the trip is:

[tex]t_{T} = 28\,h +20\,h[/tex]

[tex]t_{T} = 48\,h[/tex]

The total duration of the trip is 48 hours.

What are the conditions of floatation in a liquid?

Answers

Answer:

The conditions for a substance to float on liquid are given below:

When a substance displaces the liquid equal to its own weight.

When the upthrust given by a liquid is more than the weight of a substance.

[tex] \mathrm{Hope \: I \: helped!}[/tex]

[tex] \mathrm{Best \: regards!}[/tex]

Condition for Flotation. An object will float if the buoyancy force exerted on it by the fluid balances its weight


Which graph represents the solution set of this inequality?
10c + 5 < 45?

Answers

Answer:

see below

Step-by-step explanation:

10c + 5 < 45

Subtract 5 from each side

10c + 5-5 < 45-5

10c < 40

Divide by 10 on each side

10c/10 < 40/10

c < 4

Open circle at 4 and the line going to the left

Please help, need to find f^-1 (x)

Answers

The inverse function [tex]f^{-1}(x)[/tex] is such that

[tex]f\left(f^{-1}(x)\right) = x[/tex]

Plugging [tex]f^{-1}(x)[/tex] into [tex]f(x)[/tex] gives us

[tex]f\left(f^{-1}(x)\right) = \dfrac{3f^{-1}(x) + 3}{5f^{-1}(x) + 6} = x[/tex]

Solve for [tex]f^{-1}(x)[/tex] :

[tex]\dfrac{3f^{-1}(x) + 3}{5f^{-1}(x) + 6} = x \\\\ 3f^{-1}(x)+3=x\left(5f^{-1}(x)+6\right) \\\\ 3f^{-1}(x) + 3 = 5x f^{-1}(x)+6x \\\\ 5xf^{-1}(x)-3f^{-1}(x) = 3 - 6x \\\\ (5x-3)f^{-1}(x)=3-6x \\\\ \boxed{f^{-1}(x)=\dfrac{3-6x}{5x-3}}[/tex]

What is the simplified sum of 3x/x-4 + x-3/2x

Answers

━━━━━━━☆☆━━━━━━━

▹ Answer

-1 - 1/2x

▹ Step-by-Step Explanation

3x ÷ x - 4 + x - 3 ÷ 2x

Divide and Rewrite:

3 * 1 - 4 + x - 3 ÷ 2 * x

Calculate:

3 - 4 + x - 3/2x

-1 + x - 3/2x

= -1 - 1/2x

Hope this helps!

CloutAnswers ❁

━━━━━━━☆☆━━━━━━━

Answer:

Step-by-step explanation:

[tex]\frac{3x}{x-4}+\frac{x-3}{2x}[/tex]

Make them into common denominators. To do so, multiply by the LCM of the denominators. The LCM of the denominators is (x-4)(2x). Thus, we multiply 2x to the first term and (x-4) to the second:

[tex](\frac{2x}{2x}) \frac{3x}{x-4}+(\frac{x-4}{x-4}) \frac{x-3}{2x}[/tex]

Simplify:

[tex]\frac{6x^2}{2x(x-4)}+\frac{x^2-7x+12}{2x(x-4)} \\=\frac{7x^2-7x+12}{2x(x-4)}[/tex]

And this cannot be simplified further (you can also distribute the denominator if preferred).

find the ratio in lowest term b 750m and 1km​

Answers

Hope it helps !!!!!!!

Answer:

4m/3m

Step-by-step explanation:

1km = 1000m

750m : 1km

750m : 1000m

Reduce ratio in simplest form.

HCF of 1000 and 750 is 250

750  ÷ 250 / 1000  ÷ 250

= 4/3

Hence, the simplest form of the ratio 750m : 1km is 4m/3m

Please answer it now

Answers

Answer:

8

Step-by-step explanation:

x+37+x+37+90 = 180

2x + 74 = 90

2x = 16

x = 8

Answer:

x=8°

Step-by-step explanation:

JI is a diameter and K is on the circumference of a circle.

∴∠JKI=90°

also KJ=KI=x(say)

tan (x+37)=y/y=1=tan 45

so x+37=45

x=45-37=8°

what’s the equation of line ?
y =__x + __

Answers

Answer:

y=3/4x-2

Step-by-step explanation:

two points from graph (0-2) and (8,4)

find slope m: y2-y1/x2-x1

m=4+2/8-0

m=6/8=3/4

x=0 then y=b=-2

y=3/4x-2

Other Questions
Which website allows you to host your podcast for at a cost that includes your domain name? (Correct answer only)A. PodbeanB. BloggerC. WordPressD. HostGatorE. Buzzsprout What is the domain of F(x) = In(x)? Which statement is true regarding the graphed functions f(4)=g(4)F(4)=g(2)F(2)=g(-2)F(-2)=g(-2) Depreciation represents a .............. in the value of Fixed Assets. What are the four main components of soil?A. WaterB. SedimentC. AirD. HumusE. Bedrock When the number of units in work in process and finished goods inventories decrease, absorption costing net operating income will typically be greater than variable costing net operating income.a. Trueb. False In a doubleslit interference experiment, the wavelength is lambda=487 nm , the slit separation is d=0.200 mm , and the screen is D=48.0 cm away from the slits. What is the linear distance x between the eighth order maximum and the fourth order maximum on the screen? Which relation is a function? 22/55 of a number is what percentage of that number Can you please help me!!!!! Order the expressions from least to greatest.2^3 - 2^1 , 2^1 + 3^1 , 3^2 6. The discovery of life on another planet would tremendousexcitement among both scientists and the general population.A. deprecateB. surmiseC. raze D. engender7. To me, geese always look_, strutting around with their chests puffedout and their heads held high in the air.A. servile B. credulous c. gratuitous D. pompous8. "Are you comfortable? Can I get you something to eat? You're lookingawfully pretty today. Would you like a pillow for your head?" my brotherasked me, making me very suspicious. Why was he being so __?A. resigned B. incipient c. listless D. servile9. Dost thou take this woman to be thy lawfully wedded wife?" the preacherasked. Amused by the language, the couple couldn't help giggling.A. recalcitrant B. soporific c. archaic D. resigned10. Hoping to sound to his date, Len spent the afternoon reading theWall Street Journal, the New York Times, and some online articles.A. hardy B. credulous c. erudite D. incipient Complete the recursive formula of the geometric sequence -0.3,0.9,-2.7,8.1 A mortgage is paid off in 30 years with a total of $124,000. It had a 2% interest rate that compounded monthly. What was the principal analyse the negative impact of administered prices on the economy 1.46 mol sample of nitrogen gas at a temperature of 14.0 C is found to occupy a volume of 23.7 liters. The pressure of this gas sample is mm Hg Find the area and the perimeter of a triangle (shown below)PLZZZ -2x + 8 = -26plz help!!!!!!!!!!!!! Find f(2) given f(x) = -3x^3 + x^2 3 The following shows the monthly sales in units of six salespersons before and after a bonus plan was introduced. At 95% confidence, determine whether the bonus plan has increased sales significantly. (For the following matched samples, let the difference "d" be: d = after - before.)Salesperson After Before1 94 902 82 843 90 844 76 705 79 806 85 80